Pedagogy

Lakukan tugas rumah & ujian kamu dengan baik sekarang menggunakan Quizwiz!

Sensorimotor Stage

0-2 years; Infants start to build an understanding of the world through their senses by touching, grasping, watching, and listening. Infants develop object permanence.

Formal Operational Stage

11 + years; Children learn logical rules to understand abstract concepts and solve problems.

Preoperational Stage

2-7 years; Children develop language and abstract thought. Children begin to use symbolic play (playing pretend), draw pictures, and talk about things that happened in the past.

Concrete Operational Stage

7-11 years; Children learn logical concrete (physical) rules about objects, such as height, weight, and volume. Children learn conservation, the idea that an object, such as water or modeling clay, remains the same even when its appearance changes.

A fourth-grade teacher plans a science unit that includes two experiments, one class project, a teacher presentation, and a field trip. Which of the following is the most important advantage of the teacher's instructional unit? A. It allows students with varied learning styles to process the information using multiple sources. B. It allows the teacher to cover a larger amount of material with less time. C. It ensures the content is structured in a coherent, understandable manner. D. It gives students hands-on experience with the instructional unit.

A. It allows students with varied learning styles to process the information using multiple sources. The various learning styles activities provided the students with multiple ways to process the material in various situations.

What is the primary benefit of a teacher working with a student to think about the step-by-step causes of a problem and the possible solutions? A. It sets an example for problem-solving strategies that the student can use in multiple situations. B. It demonstrates the use of higher-order thinking skills in identifying multiple solutions to one problem. C. It demonstrates how small issues can lead to large problems and how to identify a problem's cause in order to avoid it. D. It provides a real-world context for classroom instruction and higher-order thinking skills.

A. It sets an example for problem-solving strategies that the student can use in multiple situations. Having students think through problems and their causes allows the students to develop problem-solving strategies. These strategies can apply to multiple situations.

Mr. Ramirez has overheard several students complain about the current instructional activity. One student said to another, "I am just bored with stuff. I don't understand why it's important and I don't care to know." Mr. Ramirez admits to himself that he has the same feelings toward the material and decides to get through the instructional unit as quickly as possible. Of the following, what is the most likely cause of the students' lack of interest in the material? A. Mr. Ramirez does not exhibit an excitement and enthusiasm for the material and his students have adopted his attitude towards the instructional activities. B. Mr. Ramirez has not presented the material in a coherent manner and students have become confused about the purpose and expectation of the instructional activities. C. Mr. Ramirez has not been able to relate the instructional material to the students' past experiences and the students are having trouble understanding the instructional activities. D. Mr. Ramirez has not adequately planned the instructional unit and the activities do not relate to the instructional goals or past instructional units.

A. Mr. Ramirez does not exhibit an excitement and enthusiasm for the material and his students have adopted his attitude towards the instructional activities. Communicating enthusiasm about the material is important in teaching. If the teacher is not excited about the subject matter, it is very unlikely that the students will be excited. Mr. Ramirez has communicated to his students, through his apathy, that the material is not important and is not worth learning.

A mathematics teacher gives her class a two-question clicker quiz at the end of each class period and tabulates their answers according to their mathematical understanding, misconceptions, and error patterns. If her goal is improvement in her students' mathematical proficiency, her best use of the data would be to use it to: A. inform upcoming instructional strategies. B. develop instruction that connects mathematical concepts and procedures. C. create a display highlighting mathematical vocabulary for English-language learners. D. teach students to model and solve problems in other disciplines.

A. inform upcoming instructional strategies. Data on student understanding, misconceptions, and error patterns is best used to inform instructional strategies on the same or subsequent topics.

Sara is a student who reads above grade-level and discusses ideas at a high level, but has been known to neglect class work and leave assignments unfinished when she does not see the point. She is not motivated by grades and her participation depends mostly on the subject being studied. Which of the following best describes Sara? A. intrinsically motivated by interest in the content B. externally motivated by grades C. intrinsically motivated by a love of reading D. externally motivated by teacher approval

A. intrinsically motivated by interest in the content Intrinsically-motivated students draw their motivation from the learning process. They find joy in the activity itself and learn for the sake of learning. Sara's participation seems to depend on how valuable she finds the assignment.

A third-grade classroom includes several students who are English Language Learners (ELLs). The students are reading a novel, and the teacher has introduced a list of vocabulary words on a bulletin board. The ELL students are not currently reading on a grade level. Which of the following strategies could help them to learn the vocabulary words? SELECT ALL ANSWERS THAT APPLY. A. pair the vocabulary word with a picture and simple definition B. provide translated versions of the vocabulary words C. have students read an abridged version of the novel in their native language(s) D. allow the students to listen to snippets of an audio version of the novel

A. pair the vocabulary word with a picture and simple definition The teacher is providing equal access to the students by providing cue and simple language that explains the vocabulary words. The teacher can reinforce learning by having students draw picture cues of their own and practice using the words in context. B. provide translated versions of the vocabulary words This would help students to recognize the vocabulary words by name and meaning when encountering the words in the novel.

When asking a student a question, which of the following should be the primary consideration of the teacher for response time by a student? A. the difficulty level of the question B. the number of other students who have raised their hands to answer the question C. the length of the class period D. how closely the question correlates to the instructional content being taught by the teacher

A. the difficulty level of the question This is the best answer option as more complex and difficult questions require longer periods of time for students to formulate and communicate responses.

Critical Thinking

Analysis and judgement Design an experiment, determine important part of a word problem, make a flowchart

Questioning

Asking questions to gather information Troubleshoot an issue, design a questionnaire

In the middle of a lesson on plant systems, a teacher asks students to raise their hand if a statement she says about roots is true. She reads four statements and notes how accurate the class is for each statement. This exercise is an example of: A. a self-assessment. B. a formative assessment. C. a performance assessment. D. a summative assessment.

B. a formative assessment. A formative assessment is given during a unit to inform further instruction. The teacher can use how well the students rated the statements to decide if they understand roots.

A science teacher uses labs to reinforce concepts taught in class. Students must read the lab as they participate, then turn in a written conclusion that has at least one illustration and one graph. Then, they will receive points for a class discussion about data at the end. What type of learner is the teacher targeting? A. kinesthetic B. all of the above C. auditory D. visual

B. all of the above The discussion will target auditory learners. Reading the lab, drawing a picture, and creating a graph will target visual learners. Doing the lab and working with materials will target kinesthetic learners.

Ms. Johnson has implemented small, homogeneous, intervention groups in her class to provide instruction to students who have similar needs. This allows Ms. Johnson to invite individual students to join the group to work on a specific set of skills. Which of the following would be the best use of these intervention groups by Ms. Johnson? A. to assess a specific group's content knowledge B. to provide instruction to a minority of students who have not demonstrated mastery of a specific content objective C. to provide instruction to a minority of students who exhibit a noticeable lack of interest during instruction D. to assess the content mastery of the high achieving students to ensure the instructional units do not progress faster than the students capacity

B. to provide instruction to a minority of students who have not demonstrated mastery of a specific content objective Intervention groups allow teachers to create a group based on ability. This is possible because the group is formed by invitation and is not mandatory. Forming groups based on homogeneous ability allows the teacher to re-teach specific skills or concepts the group may not have grasped during regular classroom instruction.

A fourth-grade science teacher plans an experiment where students can freely manipulate the material, compare progress, and share their conclusions with their classmates. Which of the following is best demonstrated in the teacher's instructional strategy? A. Student involvement in classroom activities increases with a decrease in classroom structure. B. Students must have peer interaction to obtain new knowledge. C. Active engagement in the instructional activities greatly increases students' learning ability. D. The students' ability to learn is greatest when they direct their own learning.

C. Active engagement in the instructional activities greatly increases students' learning ability. The experiment allows students to be actively engaged, which research suggests greatly increases students' learning ability.

Mrs. Clark has tasked her students to write a poem about a memorable time in their life. Of the following, which is the most important assessment principle Mrs. Clark should implement? A. Assessments should be informal and focus on each student's comprehension. B. Assessments should be returned within two weeks of the completed assignment. C. Assessments should be objective and concrete. D. Assessments should be varied and allow students to demonstrate content knowledge through multimedia.

C. Assessments should be objective and concrete. Every teacher should strive to have each assessment be objective, so favoritism or subjectivity cannot influence students' grades.

An elementary school just developed a new science activity center for its first- and second-grade classes. The principal wants to ensure the science center is effective. Which is the best method of evaluation for the principal to use in assessing the center's effectiveness? A. Compare the levels of school enjoyment between the students that use and do not use the center. B. Count the number of students that use the center before and after recess. C. Observe the first- and second-graders as they work with the materials and use the resources provided by the center. D. Require students who use the center to complete a survey which evaluates how well they think they know science.

C. Observe the first- and second-graders as they work with the materials and use the resources provided by the center. Observing how the students interact with the material is the best way to assess the effectiveness of the activity center.

As Mrs. Matthews, an ELAR teacher, grades a required composition, she writes a note concerning each of her students' work. According to research on instructional feedback, which elements would be the best method for providing feedback to students? A. promoting a positive learning environment by putting more emphasis on what the students did correctly rather than specifically identifying the students' mistakes B. giving both positive and negative feedback in general C. using an established rubric with further notes of the specific positives and informing the student about the specific errors that need to be addressed. D. informing the student of specific individual errors

C. using an established rubric with further notes of the specific positives and informing the student about the specific errors that need to be addressed this is the best answer choice. Students would have already been introduced to and understand that this type of rubric will be used for grading and that they will benefit from specific feedback.

Which of the following situations is an example of extrinsic motivation? A. A student is motivated to volunteer at an animal shelter because she has previously adopted a cat and wants to help other animals in need. B. A student is motivated to try out for the concert choir because he has never participated in this group and wants to learn a new skill. C. A student is motivated to implement coping strategies that he has practiced in counseling sessions because he wants to find more productive ways to deal with frustration. D. A student is motivated to complete her homework because her parents said that she can have ice cream once she is finished.

D. A student is motivated to complete her homework because her parents said that she can have ice cream once she is finished. Completing homework in order to have ice cream is an example of extrinsic motivation because the student is completing her work in order to earn a reward which can be a token or privilege.

An elementary classroom has a new student. The teacher realizes it can be a slow process for the new student to be accepted and feel welcomed in a new classroom. Which of the following strategies would best promote the class's acceptance of the new student? A. Ask each student to interview the new student, then gather the interviews to create a biography. B. Require the new student to stand in front of the class and give a brief autobiography. C. Assign the new student to help in various chores. D. Create teams of students and assign each team to help the new student learn in a classroom activity and daily routine.

D. Create teams of students and assign each team to help the new student learn a classroom activity and daily routine. This is the best option because the new student will be able to meet each student and learn the classroom activities and routines.

Which of the following is not common for a seven-year-old student? A. able to tell time B. able to recognize left from right C. able to understand and use multiple-meaning words D. able to write paragraphs

D. able to write paragraphs This is a skill that is more often developed at age nine.

Students in second grade are learning about metamorphosis. What is the best way for them to learn this concept? A. Go on a nature walk during science time on a set day each week and look for caterpillars, cocoons, and butterflies. B. Read "The Very Hungry Caterpillar" and have them discuss metamorphosis. C. Create a worksheet that directs students to color and cut out pictures and then place them in the right order. D. Have them raise a caterpillar and document each day what they observe as it forms a chrysalis and becomes a butterfly.

D. Have them raise a caterpillar and document each day what they observe as it forms a chrysalis and becomes a butterfly. This hands-on experience allows them to observe the process of metamorphosis and learn while they are documenting.

Mr. Newton, an elementary teacher, is organizing his classroom resources for the upcoming school year. One of his goals for the year is to help students appreciate and understand cultural diversity. Which of the following strategies would be most effective for meeting this goal? A. Place posters on the walls depicting various cultures. B. Create a calendar that highlights the holidays and significant celebrations of various cultures. C. Create a learning center activity about various cultures for individual students to complete independently. D. Incorporate books and other resources that represent a variety of cultural perspectives into classroom learning lessons.

D. Incorporate books and other resources that represent a variety of cultural perspectives into classroom learning lessons. This is the best answer because students can use the books and resources to gather various cultural perspectives and gain a deeper understanding of them. Multiple resources ensure that students can use multimedia to best suit their learning styles and that the content is as varied as the students' desire. Also, incorporating the content into the classroom learning lessons allows the students to gain a better grasp of the concepts.

After a long day at school, Miss Jackson gets home and opens up Facebook. She is frustrated because a parent has been complaining about her teaching methods and behavior management. Which of the following is acceptable to post as a status update on her social media page? A. "Education would be great if it weren't for parents! Such a long day!" B. "I love my job, but it can be so frustrating sometimes." C. "Can't wait until June when I don't have to deal with parent meetings." D. Nothing. As a teacher, it is unprofessional to post negative comments about education.

D. Nothing. As a teacher, it is unprofessional to post negative comments about education. The professional life of a teacher should not be discussed on social media.

Mr. Barrios is teaching a unit on multiplication to his fifth-grade class. On the very first day he gives an exit slip with the following problem on it: 123.456 x 789 = ____________ Every single student gets the question correct. How should he adjust his teaching? A. Give more difficult exit slips. B. Continue with his unit as planned; his students are clearly understanding the material. C. Reteach the lesson because students are failing to understand the material. D. Teach more advanced multiplication content to challenge his students.

D. Teach more advanced multiplication content to challenge his students. This is the best answer. While every student getting the answer to the exit correct could indicate stellar teaching, it could also indicate that students had previously mastered the material. He should consider adding more advanced content to his lessons to keep his student challenged and engaged.

From the list below, which activity is most likely to help students understand the impacts of humanity and technology on the environment? A. a discussion about safety during a natural disaster, such as a tornado B. a lesson on what happens to a person's body when they exercise regularly (exercise physiology) C. reading about the effects of relaxing a national law on the protection of an endangered species, such as the Preble's jumping mouse D. a field trip to a nearby location where humans have impacted the environment

D. a field trip to a nearby location where humans have impacted the environment Personally examining the effect of humans on a nearby location (for example, a dam in the dry west, leaves in a flood zone, or even litter in a park) allows students to point out what has been done and how technology can be both helpful and hurtful to the environment.

During a lesson on simple machines, Ms. Zhang set up levers, pulleys, ramps, and weights for her third-grade students to manipulate and observe. She noticed that the majority of students were just playing with the materials, for example, launching paper wads with the levers, rather than applying what they had learned about simple machines. What is the best thing Ms. Zhang could have started with to keep students more on task during the manipulation activity? A. step-by-step instructions telling students what to do and in what order B. a worksheet with specific questions about weights and machines for students to fill out C. a reading assignment about simple machines and demonstrations on how they work D. demonstrations on use of the machines and guidance on what to explore and why

D. demonstrations on use of the machines and guidance on what to explore and why Often a quick direct instruction with a demonstration is ideal before a lab. Students need to know what to look for and what to think about as they manipulate equipment in a science class. Inquiry does not mean simply giving materials to students and letting them explore without guidance. Demonstrations on how to use the equipment, and guiding questions such as "Lever: Explore how hard you must push to lift a weight when the fulcrum is moved closer to or farther from the weight." will help students use the time as a learning experience.

Spectator Play

Watching others play without engaging; ages 2 - 2.5 years

When teaching social studies, Ms. Linder is hoping to encourage self-directed learning. What result is she hoping to achieve with this focus? A. decrease the amount of homework for students B. transition from multiple-choice assessments to project-based learning C. increased parent participation D. increased student ownership of learning

D. increased student ownership of learning One of the by products of self-directed learning is students beginning to take responsibility for their own education and developing a sense of ownership in their learning. Students will most likely have a better learning experience and become lifelong learners if they have ownership in their education.

A teacher is monitoring her class while the students are involved in a group activity exploring the size of angles in a set of triangles. She moves from group to group, pausing and watching the group dynamics. What the teacher is doing can best be described as: A. informal summative assessment. B. formal formative assessment. C. formal summative assessment. D. informal formative assessment.

D. informal formative assessment. The teacher is listening and watching as she informally assesses student understanding of the concept: angle measures in triangles.

Physical Development

Development of the body and coordinated movement

Early Reader (Stage of Reading Development)

Early readers begin understanding that reading from the printed page needs to make sense - both from the pictures and from the print

Decoding

In reading out loud, being able to sound out words by breaking them into simple forms. In reading for comprehension, the understanding of how to read each letter or letter pattern in a word to determine the word's meaning.

Social Development

Interactions with people or the environment

Creative Thinking

Looking from a new perspective Brainstorm, have a debate

Parallel Play

Playing a similar activity alongside a peer without interaction; ages 2.5 - 3 years

Solitary Play

Playing independently or alone; ages 3 months - 2 years

Cooperative Play

Playing with peers by creating rules or guidelines for the activity together; ages 4 and up

Associate Play

Playing with peers in a loosely-organized manner; ages 3-4 years

Third-Party Perspective

Seeing form one's, others, and a third person's perspective (neutral bystander, hence sometimes called the bystander perspective); generally keeps multiple perspectives in mind at the same time Jessica loves peanut butter and jelly sandwiches. She knows that some other people don't like them, but that it's alright for two people to hold different opinions about a subject.

Reasoning and Planning

Solving problems and making decisions Create a model, demonstrate a method, make up a game

Alphabetic Principle

The understanding that there is a logical/systematic relationship between the sounds of spoken English and the letters and letter-patterns of written English.

Cognitive Development

Thinking or learning

A second-grade teacher is reading aloud two versions of the same fairy tale written by different authors. The teacher would like to support students' ability to compare and contrast the stories. Which graphic organizer would be most appropriate for this purpose? A. Venn Diagram B. Basic Outline C. Hierarchical Topical Organizer D. Bubble Topical Organizer

Venn Diagram

Alphabetic knowledge

ability to recognize, name, and write letters.

Undifferentiated Perspective

attributing one's own perspective to everyone else's; generally have an inability to attribute any other perspective to anyone else Jessica loves peanut butter and jelly sandwiches. She thinks everyone else loves them too.

Affective Domain

emotional skills appropriate reactions to situations in the classroom, like calmly asking for help when a student does not understand

Cognitive Domain

knowledge and intellectual skills multiplication, spelling

Psychomotor Domain

physical, motor skills correctly holding a pencil when writing

Emotional development

processing and understanding feelings

Early Fluent / Fluent / Proficient Reader (Stage of Reading Development)

readers recognize many words and can apply phonics and word analysis skills to figure out unfamiliar words

Self-Reflective Perspective

seeing, thinking, and feeling from the other person's perspective (first signs of empathy); generally still assumes the other perspectives are equally valid. Jessica loves peanut butter and jelly sandwiches. She knows that some other people don't like them and can begin to understand that the other person's perspective is ok.

Memory / Recall

the ability to provide facts from given information Recite a poem, make a timeline, list main events

Phonological Awareness

the understanding and ability to hear individual words, syllables, and sounds in spoken language apart from print

Social-Informative Perspective

understanding that perspectives exist and can be different for different persons; generally still assumes their own perspective is the best Jessica loves peanut butter and jelly sandwiches. She knows that some other people don't like them, but she thinks they are wrong.

Societal Perspective

understanding that the neutral bystander is influenced by the societal and cultural context; generally realizes that different societies would have different neutral perspectives Jessica loves peanut butter and jelly sandwiches and understands that other people do not. She also understands that both her opinion and the opinion of the other are influenced by larger institutions, such as family and culture.

Emergent Reader (Stage of Reading Development)

when children understand that written language has meaning and gives messages

Students in a kindergarten class are beginning a unit on the senses, and the teacher wants to implement a project-based learning experience related to this topic. Which of the following guiding questions related to the unit topic would be most suitable for this purpose? A. How do we use our senses to discover our world? B. Which sense do you think is most important? C. What parts of our bodies do our senses use? D. Can we function without one of our senses?

A. How do we use our senses to discover our world?

Which of the following teacher actions best reflects the concept of formative assessment? A. A third-grade teacher gives students a quiz midway through a social studies unit to check their understanding. B. A fourth-grade teacher drops students' lowest mathematics grade when calculating their report card grades. C. A fifth-grade teacher draws questions for an end-of-unit science test directly from the relevant textbook chapters. D. A sixth-grade teacher asks each student to submit two questions for an upcoming literature test.

A. A third-grade teacher gives students a quiz midway through a social studies unit to check their understanding. This question requires the examinee to demonstrate knowledge of the characteristics, uses, advantages, and limitations of different types of formative and summative assessments. Formative assessment is designed to monitor students' learning progress during a lesson or unit. Formative assessments can be formal or informal and provide the teacher with feedback on the effectiveness of instruction. Using formative assessment during an instructional sequence allows the teacher to modify instruction to more effectively promote student learning.

Students in a third-grade class have been studying fairy tales. They have learned common elements of fairy tales and read fairy tales from different cultures. The teacher wants to prompt students to think about fairy tales in novel ways. Which of the following assignments would best address this goal? A. Rewrite a popular fairy tale in a modern setting with a different outcome for the main character. B. Survey your classmates about their favorite fairy tale characters. C. Use a venn diagram to compare and contrast the traits of the main characters in two fairy tales. D. Create a puppet of one character in your favorite fairy tale.

A. Rewrite a popular fairy tale in a modern setting with a different outcome for the main character.

Mrs. Sims wants to teach her first-grade science class how parts work together to allow systems to work, and without all the parts, systems may not work properly. Which of the following instructional strategies is the best way to convey this concept? A. Use a system the students are familiar with as an example, such as demonstrating that a computer does not work without a power cord. B. Show a diagram of a basic system and how all of the parts work together to allow the system to function. C. Teach the concept multiple times and adjust the vocabulary used to describe key parts. D. Give students a worksheet that highlights the key terms of the instruction and have them work through the worksheet prior to the instructional activity.

A. Use a system the students are familiar with as an example, such as demonstrating that a computer does not work without a power cord. Students at the first-grade level need instruction that correlates to events, circumstances, or knowledge they are familiar with or can see. Students at this age learn best by being able to work with, observe, and replicate instructional concepts.

A test given at the end of a unit on weather and climate is an example of: A. a summative assessment. B. a formative assessment. C. a performance assessment. D. a self-assessment.

A. a summative assessment. A summative assessment is given at the end of a unit to find out what standards students have met.

Which of the following strategies is most important in providing student-centered instruction that is tailored to students' individual strengths and needs? A. engaging students in an ongoing cycle of goal setting and reflection B. adopting a flexible approach in applying evaluative criteria to students' work C. allowing students to define their own learning objectives and evaluative criteria D. relying primarily on information from peer assessments when assigning students' grades

A. engaging students in an ongoing cycle of goal setting and reflection

A second-grade teacher discovers that students' current knowledge about the topic of an upcoming social studies unit is quite limited. in this situation, the teacher's unit plants should include provisions for: A. ensuring that students have a foundation of background information on which to build new learning. B. administering formal assessments of student learning more frequently that usual during the unit. C. expanding homework assignments to provide students with additional practice on important content. D. modifying the standards on which the unit is based to accommodate students' characteristics.

A. ensuring that students have a foundation of background information on which to build new learning.

A fourth-grade teacher makes a point of welcoming individual students as they arrive each day and takes time to talk with students about their interests and activities on a regular basis. The most important benefit of this practice is that it allows the teacher to: A. establish a pattern of positive interaction with students. B. implement consisten. classroom procedures. C. integrate informal social interaction into the daily routine. D. model effective communication skills for students.

A. establish a pattern of positive interaction with students.

A sixth-grade teacher will be meeting with parents/guardians to discuss their children's scores on a recent standardized test. The teacher can best help parents/guardians gain an accurate understanding of their children's performance by taking which of the following actions prior to the meetings? A. gathering evidence of each student's classroom performance to supplement test scores B. reviewing the test administration manual to identify characteristics of the various score reports C. creating a short written summary of each student's scores to supplement the general report D. developing a comparison table between subarea test scores and classroom assessments

A. gathering evidence of each student's classroom performance to supplement test scores This question requires the examinee to demonstrate the ability to apply skills and strategies for communicating effectively with parents/guardians and others about assessment results. Supplementing a discussion of standardized assessment scores with examples of student work makes the test data more meaningful for parents/guardians. This process links the data to students' actual products and work and facilitates parents'/guardians' ability to recognize their child's specific strengths and needs.

Third-grade students have finished writing a three-paragraph essay. The teacher plans to have students read their essay to the class but is aware that some students are reluctant to read aloud. The teacher wants to accommodate these students' needs while increasing their comfort level with reading aloud. Which of the following strategies is likely to be most effective in achieving this goal? A. giving students the option of making an audio recording of their essay to play for the class B. exempting students from reading their essay until they gain confidence in their reading abilities C. allowing students to select a peer who reads fluently to read the essay with the student D. advising students to practice reading their essay at home before reading it in class

A. giving students the option of making an audio recording of their essay to play for the class

An elementary school teacher encourages students to identify a topic of interest to investigate when they have completed their work. Periodically, the teacher has students share what they are learning with the class. This approach is likely to have which of the following benefits? A. helping students become self-motivated in the pursuit of learning B. improving students' ability to synthesize information from multiple sources C. enhancing the teacher's awareness of students' learning preferences D. expanding students' use of varied problem-solving strategies

A. helping students become self-motivated in the pursuit of learning This question requires the examinee to demonstrate the ability to apply skills and strategies for establishing a culture of learning that promotes self-motivation. One important factor in students' self-motivation is their engagement with the content or task. When students are allowed to choose a topic they are interested in, they are more likely to be engaged in and develop a sense of ownership about their learning. Another important factor in students' self-motivation is their perception of the value of the learning task. In having students share what they are learning with the class, the teacher communicates to students that their learning is important and valuable.

During a routine conference, a student's mother becomes very emotional and confides that the family is struggling with economic challenges. She explains that the student's father has been looking for work for almost a year and they are in danger of losing their house to foreclosure. Which of the following would be the teacher's most appropriate response to the mother's disclosure? A. helping the family locate information about available community services and how to access them B. asking the school's Parent Teacher Student Association (PTSA) to help the family with the student's school-related expenses C. providing relevant social services agencies with information about the family's specific needs D. sending a note to the parents/guardians of other students in the class notifying them of the family's situation

A. helping the family locate information about available community services and how to access them

Students in an elementary classroom have varied learning preferences. In this situation, the teacher can best promote learning for all students by emphasizing which of the following approaches? A. implementing activities that address multiple learning modalities as a regular part of instruction B. surveying students regularly to monitor changes in their learning preferences C. gathering feedback from students about their preferred learning activities at the end of each unit D. designing learning center activities that address a different modality each week

A. implementing activities that address multiple learning modalities as a regular part of instruction

Which of the following cognitive advances typically occurs between the ages of six and nine? A. making the transition from "learning to read" to "reading to learn" B. replacing magical beliefs with plausible explanations C. identifying cause-and-effect relationships in familiar situations D. distinguishing the difference between estimating and guessing

A. making the transition from "learning to read" to "reading to learn"

In applying the principles of observational learning, the teacher's most important first step is to: A. present clear and interesting cues to engage students' attention. B. link new content to students' current knowledge. C. describe for students a positive consequence of learning a new skill. D. model the desired learning outcome for students.

A. present clear and interesting cues to engage students' attention.

Mr. MacArthur is about to assign a major project to his students. Which of the following strategies would help promote students' understanding of the project's expectations? A. providing and discussing the rubric that will be used to grade the project with the students B. establishing work benchmarks where students must present a completion of work to the instructor throughout the project C. asking students to discuss the project with their parents and allowing the parents to contact the teacher directly with any questions D. requiring students to self-assess their progress as they work to complete the project

A. providing and discussing the rubric that will be used to grade the project with the students Discussing the rubric and providing it to students ahead of time will outline the requirements and grading standards for the project.

Unpacking curriculum standards is an important strategy in helping teachers achieve which of the following instructional goals? A. providing students with integrated learning experiences B. increasing the pace at which students learn targeted content C. connecting learning activities to students' personal experiences D. using grouping strategies that will best facilitate student learning

A. providing students with integrated learning experiences "unpacking curriculum standards" : reveals the component concepts and skills associated with each standard. When these are revealed, teachers can better relate concepts and ideas across curricular areas For more information: https://blog.masteryconnect.com/four-steps-unpacking-standards/#:~:text=%E2%80%9CUnpacking%E2%80%9D%20is%20a%20technique%20teachers,targets%20to%20make%20them%20actionable.&text=As%20you%20teach%20and%20report,to%20students%20and%20community%20stakeholders.

Students in a fourth-grade class like to use word processing software for their assignments whenever possible. However, they often become frustrated by formatting issues and are sometimes unhappy with how their finished work looks. The teacher can best help students overcome these problems by implementing which of the following strategies? A. providing students with templates to use for different types of assignments B. asking the school technology specialist to present a lesson on using the software C. teaching students how to highlight and make text changes in their assignments D. ensuring that a software user's guide is located near each classroom computer

A. providing students with templates to use for different types of assignments

Fourth-grade teachers have developed several new social studies lessons. Each teacher volunteers to implement one of the lessons first in his or her classroom while the other teachers observe. The most important advantage of this process is that it enables the teachers to: A. refine elements of the lessons to better address students' needs. B. link lesson concepts more closely to students' background knowledge. C. identify the tools that would be most effective in assessing students' progress. D. gain a better sense of how teacher collaboration improves student learning.

A. refine elements of the lessons to better address students' needs.

which of the following skills is typically the last to emerge for English Learners? A. understanding academic language B. questioning to check for understanding C. self-correcting spoken errors D. following verbal instructions

A. understanding academic language

Mr. Prince teachers fourth-grade math. He records his lessons and posts them online for students to watch when they need to review. He explains verbally and writes down instruction. He also offers manipulatives for the class to use when needed. Mr. Prince's way of teaching is a great example of: A. using varied strategies and adjusting communication to reach all students. B. working too hard to teach simple concepts. C. using technology to teach students. D. visual teaching style.

A. using varied strategies and adjusting communication to reach all students. Students learn in different ways (for example: visual, auditory, tactile) and teachers should try to present concepts with explanations in the varying modalities of learning to maximize student engagement and learning success. Having students listen, write, stand, and interact with the subject matter in order to ensure understanding.

In which of the following classrooms is a teacher best promoting students' understanding of how technology can be used as a tool for learning? A. Kindergarten students listen to audio recordings of alliterative poems during their daily rest periods. B. First-grade students use the Internet to locate real-time weather data for their community and a community in another region to compare climate differences. C. Second-grade students are allowed to use drawing software on classroom computers when they finish assignments early. D. Third-grade students as a class visit the school computer center each week to practice basic multiplication facts.

B. First-grade students use the Internet to locate real-time weather data for their community and a community in another region to compare climate differences. This question requires the examinee to demonstrate knowledge of how digital-age tools and environments influence learning processes and outcomes. The first-grade students are using the Internet to actively participate in a purposeful and complex learning experience that involves higher-order thinking. Embedding technology into the weather assignment helps students view the technology as another tool, similar to a thermometer or rain gauge, that they can use to help them gather and use information to accomplish a particular task.

In which of the following situations would it be most appropriate to provide Tier 2 (i.e., targeted) instruction for a student in a core curricular area? A. Pre-assessment data indicate that the student is in need of additional practice on foundational concepts. B. Following differentiated instruction, post-assessment data indicate that the student still does not meet grade-level expectations. C. Intensive interventions have proven to be ineffective in helping the student achieve grade-level expectations. D. The student's parents/guardians request additional support to help the student attain a higher level of academic achievement than his or her current level.

B. Following differentiated instruction, post-assessment data indicate that the student still does not meet grade-level expectations. This question requires the examinee to demonstrate knowledge of the foundational elements of Response to Intervention (RtI). According to the Indiana Response to Instruction model, Tier 1 consists of providing all students with differentiated instruction in the core curriculum and then assessing their mastery. If, at this point, groups or individual students have not mastered the targeted concepts or skills, then a Tier 2 intervention such as additional small-group instruction is implemented to address students' areas of need.

During the summer, a teacher obtains the standardized test results of the previous year for her upcoming students. This information can be best used to: A. Discuss the results with other teachers to plan interdisciplinary units. B. Identify the instructional needs of each student and plan instruction to address gaps. C. Identify students with similar struggles in order to place them into groups. D. Identify the best students in the upcoming class and plan instructional units to meet their needs.

B. Identify the instructional needs of each student and plan instruction to address gaps. Using the results from last year gives the teacher an up-to-date profile of the students' academic capabilities and needs. Using this, the teacher can identify potential weaknesses or areas where students need further instruction.

Mr. Marks wants to create a class-wide research project for his third-grade class about the school's water usage and how the school can conserve their resources. As he discusses the project, the students become excited about being able to research various aspects of the school. Mr. Marks decides to split the class into small groups with each group coming up with ideas to implement the research. Which of the following is the most important advantage of incorporating student input into the planning of the project? A. It encourages the parents to assist their child in the learning process. B. It allows students to help structure and determine their own learning. C. It allows Mr. Marks more time to plan instructional units of future instruction. D. It allows students to recognize their own academic needs regarding research techniques.

B. It allows the students to help structure and determine their own learning. Providing the students an opportunity to give their own input allows them to help in the planning process of their learning. This is important because students are more likely to be involved and take ownership if they have influence in the planning.

Mr. Martin is planning an instructional unit and wants to adjust the lesson to meet the needs of the English Language Learners (ELL) in his class. Each student has a varying level of English proficiency. What is the best strategy to adjust the instructional unit to meet the student's needs? A. Send the ELL students to the resource room for supplemental instruction during presentations. B. Make dictionaries accessible to the students and allow for additional time to complete assignments while other students can work independently. C. Allow the ELL students to work together during group activities to promote English proficiency. D. Adjust the content to make the presentation and homework assignments simpler to accommodate varying language proficiencies.

B. Make dictionaries accessible to the students and allow for additional time to complete the assignments while other students can work independently. By allowing students a dictionary and additional time, the teacher is adjusting instruction to meet the needs of the ELL students while promoting English proficiency and grade-level appropriate content.

Which of the following would most help students develop time-management skills while working on a multi-step project that extends across many weeks? A. Require students to journal their progress and describe their next steps at the end of each class period. B. Provide calendars to the students with the project due date marked, then ask them to create a plan for when each part of the project should be completed along the way. C. Allow students to create and have access to a computer folder to store and maintain their research notes and outlines. D. Ask the librarian to demonstrate to students how best to maintain research notes through a research project.

B. Provide calendars to the students with the project due date marked, them ask them to create a plan for when each part of the project should be completed along the way. Providing calendars and giving students the responsibility of setting a timetable within the project will encourage time-management skills and give students more ownership over their learning.

Which of the following statements describes the most likely outcome when teachers consistently exhibit enthusiasm for the content they are teaching? A. The amount of class time that students are engaged in learning will increase. B. Students will view the content as important and valuable. C. The amount of class time lost to student behavior problems will decrease. D. Students will master the targeted content quickly.

B. Students will view the content as important and valuable.

A student is most likely to demonstrate a high level of motivation for and engagement in a learning task under which of the following conditions? A. The task requires minimal effort by the student for successful completion. B. The student perceives the task to be interesting and useful. C. The task requires the student to apply learned concepts and skills. D. The student anticipates a reward upon successful completion of the task.

B. The student perceives the task to be interesting and useful

Mr. Appleton is considering providing some material for his students from the internet. Which of the following is Mr. Appleton legally permitted to do under fair use guidelines? A. Compile interesting articles and videos from various websites for non-instructional use by the students. B. Use pictures and information from various websites in a class presentation. C. Distribute a single-user access login, purchased by the school, to students. D. Stream an entertaining movie from a website as a reward to students for good grades.

B. Use pictures and information from various websites in a class presentation. Using parts of various websites for instructional purposes is allowed under fair use guidelines.

The mastery learning approach to instruction is based on which of the following principles? A. All students possess certain innate abilities that serve as a basis for learning. B. With sufficient time and appropriate instruction, all students can attain lesson and unit objectives. C. All students possess a similar desire for success and validation as learners. D. With access to sufficient resources, all students can attain high levels of academic achievement.

B. With sufficient time and appropriate instruction, all students can attain lesson and unit objectives.

Ms. McManus has purposefully set a climate of high expectations in her classroom. Which of the following are NOT likely to be seen in her class? A. a sign that says "good students are persistent students" B. a sign that says "good students are A students" C. a behavior contract that has been written by students and is signed by all in the class D. a list of goals at each students' learning space, specific to the student

B. a sign that says "good students are A students" While making all A's is a good goal for any student, the grade earned is not the only marker of a climate of high expectations. Building a climate of high expectations does not rely only on earning A's.

Mr. Redman is teaching his eighth-grade social studies class about the Oregon Trail and the difficulties early pioneers faced when moving from the eastern part of the United States to the west. To help his students understand the difficulty of the journey, he has them play a computer game that allows them to begin in St. Louis and try to move their family west along the Oregon Trail. This type of learning activity would best be described as: A. a group activity. B. a simulation activity. C. an assessment activity. D. a memorization activity.

B. a simulation activity. The students are playing a computer game that simulates the obstacles and trials of the Oregon Trail. This is the correct answer.

A sixth-grade science teacher regularly uses tools such as idea webs and Venn diagrams to support content instruction. This practice is likely to be most effective in facilitating students' ability to: A. conduct research efficiently B. analyze relationships between concepts C. formulate questions for inquiry tasks. D. apply learned concepts in new contexts.

B. analyze relationships between concepts

A second-grade teacher has established a goal of improving students' reading fluency. The teacher is considering ways to encourage students and their parents/guardians to read together on a regular basis. Which of the following is likely to be the most effective first step for the teacher to take in achieving this outcome? A. asking students' parents/guardians to keep a log of the about of time they read with their children at home B. creating a classroom collection of students' favorite books that they can take home and share with their parents/guardians C. providing students' parents/guardians with information about children's reading activities at local libraries D. scheduling conferences with students' parents/guardians to discuss strategies for guiding students during reading

B. creating a classroom collection of students' favorite books that they can take home and share with their parents/guardians

Within the context of curriculum mapping, decisions to change or modify instructional practices should be made only when: A. teachers have received relevant training. B. data-based proof exists to do so. C. administrator approval is provided. D. consensus among teachers is achieved

B. data-based proof exists to do so.

Teachers in an elementary school are organized into professional learning communities. Which of the following teacher practices is most essential to the success of these learning communities in promoting student learning? A. participating in ongoing training in conflict-resolution strategies B. engaging in collaborative decision making based on student data C. adopting a team approach to providing classroom instruction D. identifying individual students' talents as early as possible

B. engaging in collaborative decision making based on student data This question requires the examinee to demonstrate the ability to participate effectively and productively as a member of a professional learning community. The primary purpose of a professional learning community is to enhance teachers' effectiveness as professionals and improve learning outcomes for all students. When teachers review student performance data as a team, they are able to identify strengths and weaknesses in student learning and begin to discuss what is working instructionally and what is not working. This in turn leads to discussions of new strategies to implement to help students learn and achieve.

A third-grade teacher notifies a student's parents that their child needs focused reading intervention to improve comprehension skills. The parents express concern that their child might miss other important instruction due to this intervention. In responding to these concerns, the teacher's most important responsibility is to: A. provide the parents with copies of materials their child will use in the intervention. B. explain to the parents how the intervention will be integrated into their child's reading instruction. C. reassure the parents that their child will have an opportunity to make up any missed activities. D. encourage the parents to visit the classroom to observe their child during reading instruction.

B. explain to the parents how the intervention will be integrated into their child's reading instruction.

Mr. Valdez planned a revision lesson where he shows the class before and after examples of the rough and final draft of an essay. He then provides class time for students to begin revising their work, but many students are emphatic that their essay is done and want to turn it in as is. Mr. Valdez backs up and models the process of revision, and writes a list on the board, specifying what to look for as they revise. What does Mr. Valdez demonstrate in this scenario? A. a willingness to allow students to learn from their mistakes B. flexibility and the ability to adjust a lesson that is not working as intended C. cross-curricular collaboration D. the ability to empathize with his students

B. flexibility and the ability to adjust a lesson that is not working as intended Modeling the process of revising and setting clear, specific expectations would allow Mr. Valdez to have concrete steps for the students to process through before turning the paper in. This will enable him to require specific steps before accepting a draft. Modeling the process of revising and setting clear, specific expectations would allow Mr. Valdez to have concrete steps for the students to process through before turning the paper in. This will enable him to require specific steps before accepting a draft.

Which of the following teacher strategies is likely to be most effective in supporting the development of third graders' time-management skills? A. ensuring that the daily schedule and clock are visible to students from anywhere in the room B. helping students establish a habit of recording assignments and projects on a planning calendar C. using a signal, such as a bell or snippet of music, to alert students when an activity is about to end D. giving students an opportunity to begin homework assignments during the school day

B. helping students establish a habit of recording assignments and projects on a planning calendar

A kindergarten teacher has a goal of promoting students' global awareness. Which of the following approaches would best support this outcome? A. providing students with explicit instruction on concepts such as diversity and tolerance B. incorporating literature, songs, and games from various world cultures into students' learning experiences C. displaying maps of Indiana, the United States, and the world prominently in the classroom D. creating a bulletin board display that features information about a different country each week

B. incorporating literature, songs, and games from various world cultures into students' learning experiences

In general, students with specific learning disabilities are more likely than their peers without disabilities to develop: A. personalized coping strategies. B. negative academic self-concepts. C. self-soothing habits such as rocking. D. obsessive-compulsive characteristics.

B. negative academic self-concepts.

Which of the following skills is most important to emphasize first in helping kindergarten students develop emotional intelligence? A. using "I" messages when communicating feelings B. recognizing and labeling feelings in themselves and others C. expressing empathy for others' negative feelings D. identifying creative outlets for expressing their feelings

B. recognizing and labeling feelings in themselves and others

After reading Jack and the Beanstalk to students, a third-grade teacher asks students the following series of questions about the story. - Did Jack trespass or go into someone's house where he did not belong? - Did Jack steal the goose that lays golden eggs? - Did Jack refuse to give back what did not belong to him? - Did Jack escape down the beanstalk and cause the giant to be killed? - If jack trespassed, stole, and caused the giant to be killed, why is the giant the villain? This line of questioning, culminating with a question designed to create conflict and confusion, is most likely to help students develop which of the following skills? A. decision making B. reflective thinking C. comprehension monitoring D. hypothetical thinking

B. reflective thinking This question requires the examinee to demonstrate knowledge of strategies for promoting students' independent thinking and learning, reflection, and higher-order thinking; and the ability to use these strategies to promote students' growth as learners. Reflective thinking is a part of the critical thinking process referring specifically to the processes of analyzing and making judgments. By asking students a "why" question that appears to be in conflict with accepted interpretations of Jack and the Beanstalk, the teacher prompts students to analyze the events of the story and make a judgment about Jack's character.

A second-grade teacher provides students with many hands-on activities related to the concepts presented in a lesson about triangles. This approach is particularly appropriate for second graders because students at this age level generally: A. are reluctant to ask questions during instruction. B. require concrete support to facilitate cognitive processing. C. need adult guidance in selecting learning activities. D. have limited background knowledge to support the learning process.

B. require concrete support to facilitate cognitive processing (Piaget)

Giving students in the upper elementary grades regular opportunities to participate in rule-based games is likely to be most effective in promoting students' development in which of the following areas? A. self-assessment skills B. social problem-solving skills C. metacognitive skills D. abstract reasoning skills

B. social problem-solving skills

A first-grade teacher is introducing the concept of wants and needs. The teacher has explined the concept to students and has written a list of common wants and a list of needs on the board, but the students are having difficulty grasping the concept. The teacher can best promote students' understanding by adapting instruction in which of the following ways? A. breaking future lessons into several short segments B. using a different strategy and familiar examples to present the concept C. extending the guided-practice portion of each lesson D. explaining the concept again using simplified language and a slower pace

B. using a different strategy and familiar examples to present the concept

Which of the following statements best describes current thinking about second-language learning without instruction during childhood? A. Children possess an innate ability to learn a second language that adults do not possess. B. Second-language competence decreases dramatically as children enter adolescence. C. Childhood is a sensitive period of development when children are particularly responsive to second-language learning. D. Neural pathways in the brain's language centers are forming during childhood, facilitating second-language learning.

C) Childhood is a sensitive period of development when children are particularly responsive to second-language learning.

Mrs. Dobbs is teaching students to skip-count by 2s, 5s, and 10s in her second-grade class. Earlier in the year, she evaluated her students learning style and assigns them one task based on this evaluation. Visual learners have been given a number line and they are to draw the hops across the top. Auditory learners have been given a list of the even numbers to 20, numbers divisible by 5 to 50, and numbers ending in 0 up to 100. They are told to say them over and over aloud to memorize the skips. Kinesthetic learners have been given a large number line on the floor. They are jumping to the next number as they skip-count. What can Mrs. Dobbs do to improve her teaching? A. Allow for more creativity by letting students make an art project out of drawing or write a song as they recite numbers. B. Allow students to choose which activity they participate in. C. Allow all students to participate in all three activities by rotating through them. D. Require them to participate in the jumping activity first to burn off some energy and then choose one of the other activities.

C. Allow all students to participate in all three activities by rotating through them. This encourages students to exercise different parts of their brain and strengthen different learning styles.

If Mr. Melton desires to evaluate the learning environment in his classroom, which of the following would be the most important question to ask himself? A. Have I fully covered the state standards assigned to my subject area? B. Do I engage my students in thinking critically and consistently use appropriate wait time? C. Do I consistently address all my students' intellectual involvement and active learning preferences when I design my lessons? D. Is my personal philosophy toward instruction and continuing education in touch with students of the 21st century?

C. Do I consistently address all my students' intellectual involvement and active learning preferences when I design my lessons? This is the best answer choice, because without peer collaboration and different levels of effective intellectual stimulus, students will not be engaged in active learning.

A team of third-grade teachers is evaluating instructional plans for an upcoming social studies unit. Which of the following questions would best help the teachers determine whether the unit is appropriately rigorous? A. Does the unit include activities that prompt students to use the full taxonomy of thinking skills? B. Are the unit activities free of topics that might prompt an emotional response from students? C. Does the unit include inquiry-based activities that require students to form their own answers? D. Are the unit activities designed to help students achieve multiple academic standards?

C. Does the unit include inquiry-based activities that require students to form their own answers? Rigor can be described as a quality of instruction that prompts students to construct meaning and impose structure to a situation rather than find meaning and structure that is already apparent.

Ms. Babu feels her class has a good understanding of the definitions and theories associated with physical and chemical changes, but she wants them to apply the concepts. Which homework assignment would best help students connect the concepts to their everyday lives? A. Have students ask their parents for the definitions of chemical or physical changes, and the next day, tell the class what their parents said. B. Have students write the definitions and one example of each of chemical and physical changes. C. Have students look for a chemical or physical change they see after they leave school, and then tell the class about it the next day. D. Have students read about examples of chemical of physical changes in daily life.

C. Have students look for a chemical or physical change they see after they leave school, and then tell the class about it the next day. Once students have a good idea of a scientific concept, asking them for examples from their own lives will motivate and help them apply the idea outside of school.

Students in a sixth-grade language arts class have finished reading John Steinbeck's "Travels with Charley: In Search of America", a memoir of his journey across the United States with his dog The teacher wants to encourage students' higher-order thinking about the text. Which of the following questions would be most effective for this purpose? A. Why do you think Steinbeck decides to take charley instead of a human on his journey? B. What reasons does Steinbeck give for embarking on his journey? C. If you followed Steinbeck's route today, what do you think you would see along the way? D. What is the nature of the relationship between Steinbeck and Charley?

C. If you followed Steinbeck's route today, what do you think you would see along the way?

Mrs. Buff receives a letter from the mother of one of her fourth-grade science students about the amount of time it is taking to do the homework for her class. Mrs. Buff has been working with all her students on developing appropriate study skills, including setting timelines for homework. Mrs. Buff responds to the letter from the parent with an invitation to the mother and her husband to meet with her concerning the issue. Which of the following would be the best strategy for Mrs. Buff to begin the conference? A. Ask the parents how much time their child spends studying each night. B. Ask the parents how much homework their child has in other classes. C. Listen to the parents' concerns and then explain the study skills that have been taught and discussed in her classroom. D. Explain they are the only parents who have ever complained about homework.

C. Listen to the parents' concerns and then explain the study skills that have been taught and discussed in her classroom. This is the correct answer because the best way to approach these parents would be to listen first and then explain.

Rather than give a unit test, Mrs. Kirby decides to assign a major project to her students. They are provided a rubric that sets the expectations and guidelines. Students will be given 2 class periods to work on it and the rest must be completed at home. Students will then present their projects in class. What is the main advantage to giving a project rather than a test? A. There are usually fewer answers to grade on a project than a test so it saves time. B. It lowers students anxieties as they do not have to prepare for a test. C. Projects require higher level thinking and can demonstrate greater concept mastery than tests. D. Parents can help their children with the project and parental involvement is key to academic success.

C. Projects require higher level thinking and can demonstrate greater concept mastery than tests. Projects with clear guidelines require students to think on a higher level and display mastery in a variety of ways.

Which of the following students is most clearly demonstrating characteristics associated with high-ability learners? A. a kindergartener who prefers looking at books to other activities and is beginning to read simple sentences B. a second grader who has well-developed listening skills and requires little guidance during independent activities C. a fourth grader who has an extensive vocabulary and masters new content with minimal repetition or practice D. a sixth grader who likes to socialize and chooses small-group activities over independent work whenever possible

C. a fourth grader who has an extensive vocabulary and masters new content with minimal repetition or practice This question requires the examinee to demonstrate knowledge of types of exceptionalities, including high ability and twice exceptional, their characteristics, and their implications for development, teaching, and learning. According to Dr. Linda Silverman, founder of the Gifted Development Center, two characteristics of gifted children are a more extensive vocabulary and memory and the ability to learn more rapidly than other children of the same age.

Which of the following is the correct definition of scaffolding? A. breaking words into small parts to sound it out B. understanding rules that govern the construction of words into phrases or sentences C. building on skills that a student already has D. hearing the different sounds of human speech

C. building on skills that a student already has Giving students a solid background/structure and extensive help on what they are presently learning is scaffolding. In this way, students should be able to build on these new skills in order for additional learning to take place without such support.

A fifth-grade class includes a few students who religious practices have clear guidelines for interactions between male and female students. In this situation, the most appropriate and responsive approach for the teacher to adopt is to: A. meet with the students' families to reach a compromise between religious practices and classroom practices. B. conduct research about the students' religion to better understand the rationale for specific gender expectations. C. consider the students' religious practices when arranging classroom seating and assigning students to small groups. D. interview the students to learn which classroom practices may be deemed inappropriate by their religion.

C. consider the students' religious practices when arranging classroom seating and assigning students to small groups

A student with disabilities is enrolling in a new school. In order to prepare for this new student, the school should understand the legal rights of the student and the responsibility of the school to: A. edit the student's IEP each grading period, based on their performance. B. ensure the student is able to make passing grades in reading and math at the same level as their grade level. C. deliver instruction to the student in the least restrictive environment. D. allow the student to be grouped with students that have similar abilities.

C. deliver instruction to the student in the least restrictive environment. This is the correct response.

Teachers in an elementary school want to improve their teaching practices to better promote students' mathematics learning. They have arranged to observe instruction in each other's classrooms on a regular basis and discuss their observations afterward. The teachers can best help ensure the effectiveness of this process by: A. keeping the school principal informed about their progress. B. choosing a mediator to assist in resolving differences of opinion. C. demonstrating receptiveness to constructive feedback. D. work toward developing similar teaching styles.

C. demonstrating receptiveness to constructive feedback

Which of the following technologies would best facilitate student learning in the sciences? A. computer scanner B. presentation software C. digital probe D. media player

C. digital probe

Which of the following features of the elementary school environment is likely to have the most significant positive influence on students' learning? A. an active group of community and parent/guardian volunteers B. frequent opportunities to interact with the school principal C. effective classroom management by the teacher D. a broad range of extracurricular opportunities

C. effective classroom management by the teacher

A DVD would be a particularly effective instructional resource for which of the following purposes? A. introducing students to new content-area vocabulary B. promoting students ability to make interdisciplinary connections C. teaching students the sequence of steps in a process D. supporting students' ability to organize information

C. teaching students the sequence of steps in a process

Which of the following assignments would be most effective in promoting third graders' higher-order thinking? A. locating a list of cities that have hosted the Olympic Games B. measuring the daily growth of a bean plant C. using the information in a bar graph to answer word problems D. answering a survey about favorite foods

C. using the information in a bar graph to answer word problems

A fourth-grade teacher routinely helps students use technology to access global information sources, shows them live streaming of selected world events, and arranges for them to use Web cameras and voice communication services to interact with students in other parts of the world. These strategies are likely to be most effective in achieving which of the following outcomes? A. promoting students' understanding of their role in a global society B. fostering in students a positive attitude toward the learning process C. enhancing students' sense of participation in a global community D. increasing the relevance of students' learning to their personal experiences

C. enhancing students' sense of participation in a global community This question requires the examinee to demonstrate the ability to provide developmentally appropriate learning experiences that promote students' sense of participation in a global community. the teacher's use of global information sources, streaming video, and voice communications serves to expand students' global awareness and provides them with a sense of connectedness to others. The teacher's strategy of arranging video interactions with students in other parts of the world is particularly effective in helping students recognize that people around the world share many common interests and aspirations.

A fourth-grade teacher is planning to implement project-based learning in science and social studies. Students have had limited experience with this approach and the teacher wants to help them be successful. The teacher can best promote this outcome by: A. assigning students a short independent study before engaging them in a full-scale project. B. leaving the time frame for a project largely open-ended with few hard and fast deadlines for students' work C. ensuring that students have the relevant research skills and content knowledge required for a project D. requesting that each student submit a few suggestions to the teacher before selecting a topic for a project.

C. ensuring that students have the relevant research skills and content knowledge required for a project This question requires the examinee to demonstrate the ability to develop and implement project-based learning experiences that guide students to analyze the complexities of an issue and use creative thinking and innovative approaches to solve problems. Project-based learning requires students to use research and inquiry skills as well as communication and presentation skills. For students to be successful with such projects, it is necessary for them to have foundational information literacy skills such as locating, analyzing, and synthesizing information. Students also need to have a body of background knowledge in the selected content area. When students have adequate foundations in these areas, they will be more prepared for the cognitive demands of project-based learning and more likely to benefit from the process.

As part of a Tier 2 intervention in mathematics, three second-grade students will receive small-group instruction to improve their addition and subtraction computation skills. The teacher can best ensure that the intervention is achieved the intended outcome by using which of the following strategies? A. collaborating with the students to develop individual learning goals B. assigning peer partners to help the students complete enrichment activities C. monitoring and documenting the students' progress frequently D. helping the students determine their preferred learning modality

C. monitoring and documenting the students' progress frequently

Students with conduct disorder are most likely to demonstrate which of the following characteristics? A. alternative periods of depression and euphoria B. auditory hallucinations and disorganized behavior C. persistent patterns of antisocial and aggressive behavior D. excessive feelings of fear and generalized uneasiness

C. persistent patterns of antisocial and aggressive behavior

Students in a sixth-grade class are reading a novel. The teacher engages the students in discussions about their reading but observes that students' responses to questions and comments tend to be superficial. The teacher wants to encourage students to consider their reading in greater depth. Which of the following questioning strategies during discussions would be most effective in promoting this outcome? A. asking a series of questions that gradually increase in difficulty B. using convergent questions to narrow the focus of the discussion C. posing probing questions that allow for multiple responses D. decreasing the number of questions posed during the discussion

C. posing probing questions that allow for multiple responses

A fifth-grade teacher develops a series of language arts lessons he believes might be useful for other teachers in the district. The teacher wants to share the lessons and invite feedback about how other teachers implement the lessons in their classrooms. Which of the following approaches is likely to be most effective for accomplishing the teacher's goal? A. asking the school principal to distrubute the lessons to other schools B. gathering e-mail addresses of interested teachers at the next district meeting C. posting the lessons and related materials on an interactive blog D. seeking leave time to discuss the lessons in person with teachers at other schools

C. posting the lessons and related materials on an interactive blog

A number of students in a sixth-grade language arts class read well below the grade level. The class will be studying three literature selections during the course of the semester. In this situation, the teacher can best apply the principles of UDL by taking which of the following actions? A. addressing different concepts and skills with students who find the literature selections too challenging B. partnering students who read below grade level with proficient readers for literature activities C. providing students with access to digital versions of the literature selections with text-to-speech capability D. adopting an open-ended approach to literature assignments regarding due dates

C. providing students with access to digital versions of the literature selections with text-to-speech capability UDL: multiple means of representation (the way information is presented), multiple means of action and expression (the way students show their understanding / the work they do), and multiple means of engagement (different ways to hook and interest students in the lesson) This answer is an example of another means of representation (the way the information in the book is presented: audio and reading)

Which of the following instructional practices is most likely to communicate high expectations for student learning? A. identifying at least one major assignment during each grading perieod that will not be graded B. developing a routine for calling on each student in the class every day C. returning students' work that is not up to standards with specific suggestions for improvement D. tailoring students' goals according to their current level of achievement

C. returning students' work that is not up to standards with specific suggestions for improvement

A first-grade teacher has led students through a number of activities in which they practiced locating information sources on a specific topic. The teacher would like to take students a step further by helping them evaluate the information they found. The teacher can best promote this outcome by helping students develop the habit of considering which of the following questions first? A. Is this information easy for me to read? B. Does this information represent a fact or an opinion? C. Is this information provided by an expert? D. Does this information answer the question I have?

D. Does this information answer the question I have?

Which of the following statements describes the most important consideration to keep in mind when communicating through e-mail or text messages? A. The sender and the receiver of a message may not have similarly developed communication skills. B. The lack of specific language conventions for online formats can result in messages that are incomprehensible. C. The speed at which messages are communicated online discourages thoughtful interactions. D. The inability to use facial expressions and tone of voice to help interpret messages can lead to misunderstanding.

D. The inability to use facial expressions and tone of voice to help interpret messages can lead to misunderstanding.

Mr. Ray typically begins each day with direct-teaching instruction where he uses the whiteboard and lecture notes to teach the students. Many times, he has turned around from writing on the whiteboard to find students distracting each other. Of the following, what would be the most effective technique to maintain the students' interest? A. Place a mirror above the whiteboard so he can monitor the students' behavior while he writes. B. Allow the students to invite local community members to speak about the content and how it relates to the students' community. C. Require the students only to have a pencil and paper on their desk. D. Use short multimedia presentations such as PowerPoint to deliver content that is accompanied by pictures or short videos

D. Use short multimedia presentations such as PowerPoint to deliver content that is accompanied by pictures or short videos. This is the best option. A multimedia presentation would be more entertaining to students and increase the chances of students paying attention during classroom instruction.

A teacher is teaching a class that all of our food comes from the sun through a series of energy transfers. Which of the following supports would best enhance a visual learner's ability to explain the energy transfers that occur during this process? A. a play students can use to act out the series of energy transfers B. a guest speaker who can tell about "farm-to-table" C. an engaging story detailing the process of the energy transfers D. a pictorial diagram using arrows to indicate the series of energy transfers

D. a pictorial diagram using arrows to indicate the series of energy transfers A pictorial diagram is a visual support that helps visual learners make sense of material.

Mrs. Hogan is wrapping up a unit on homeostasis. She would like to assess student knowledge before she moves on. Which of the following would be the best summative assessment? A. a review game over homeostasis B. a lab activity in which students observe homeostasis and the process required to maintain it C. a video and accompanying worksheet over homeostasis in animal cells D. a test over homeostasis and the processes involved

D. a test over homeostasis and the processes involved Tests are summative assessments that allow the teacher to reflect and be sure there are no large gaps in knowledge.

Teachers in an elementary school are planning a multimedia presentation for an assembly during School Safety Week. To help ensure that the presentation is effective in helping students learn the targeted safety concepts, the teachers should adhere to which of the following guidelines when designing the presentation? A. varying the color scheme used in each slide B. minimizing the use of animation to illustrate content C. using a variety of fonts to maintain interest D. avoiding the use of extraneous words, pictures, and sounds

D. avoiding the use of extraneous words, pictures, and sounds Extraneous: irrelevant Only use words, sounds, and pictures that are relevant to content and do not overdue the amount that you use.

To promote students' ability to function effectively in emerging digital learning environments, teachers should place the greatest emphasis on helping students develop and practice their skills in which of the following areas? A. project management B. reflection and self-assessment C. creative problem solving D. communication and collaboration

D. communication and collaboration

Differentiated instruction is characterized primarily by which of the following processes? A. using fixed-ability groups to efficiently address differences in achievement levels B. implementing a program of learning experiences that address increasingly complex content C. seeking feedback from students on the effectiveness of various teaching strategies D. creating learning experiences that are responsive to students' varied interests and needs

D. creating learning experiences that are responsive to students' varied interests and needs

Which of the following approaches represents the most effective way to differentiate instruction for a class of fifth-grade students who perform at a wide range of skill levels? A. revising academic standards to accommodate students' individual skill levels B. acquiring the textbooks used at lower or higher grade levels as appropriate for students' needs C. implementing small-group activities in which each group includes students at various skill levels D. designing tiered assignments for students that address the essential concepts being taught

D. designing tiered assignments for students that address the essential concepts being taught

Several times a month, employees from neighborhood businesses volunteer in kindergarten classrooms to help students with targeted skills related to reading readiness. When planning the lessons that will include the services of volunteers, the teachers should make which of the following tasks their most important priority? A. arranging for the business owners to visit the classroom when volunteers are working with students B. providing the volunteers with information about students' preferred learning modalities C. documenting for business owners the hours volunteers are actively engaged with students D. ensuring that the volunteers understand the purpose of the learning activities and how to implement them

D. ensuring that the volunteers understand the purpose of the learning activities and how to implement them

An elementary school teacher can best provide relevant and responsive instruction by taking which of the following actions before beginning the planning process? A. introducing students to a variety of content-area resources B. observing informally students' interactions with their peers C. reviewing students grades and samples of their previous work D. examining carefully students' characteristics and experiences

D. examining carefully students' characteristics and experiences

A parent expresses concerns to a teacher about a student in the class he believes may be experiencing neglect at home but is unsure of how to address the issue. Which of the following actions represents the teacher's legal and ethical obligation in this situation? A. asking school staff to begin documenting signs of neglect they observe regarding the student in question B. offering to take responsibility for reporting the concerns to Indiana child welfare authorities on the parent's behalf C. notifying the parents/guardians of the student in question that another parent has raised concerns about their child's well-being D. explaining to the parent that Indiana law requires individuals to report suspected abuse or neglect to the relevant authorities

D. explaining to the parent that Indiana law requires individuals to report suspected abuse or neglect to the relevant authorities

An elementary school holds an annual back-to-school picnic for teachers, students, and parents/guardians before the first day of classes. The most important benefit of this type of school-sponsored event is that it enables teachers to: A. observe students' behavior in a naturalistic social setting. B. discuss with parents/guardians the teacher's goals for student learning C. maintain relationships with former students and their families. D. gain insight into the broader school community in which students participate.

D. gain insight into the broader school community in which students participate.

Which of the following is the primary reason that teachers administer formative assessments? A. comparing student performance to that of same-aged peers across the country B. assessing student performance at the end of a chapter of study C. determining current knowledge before beginning a unit of study D. identifying mastered skills and skills in need of further development

D. identifying mastered skills and skills in need of further development A formative assessment is used to provide feedback on students' progress and identify which areas in which students need further instruction so that teachers can develop lesson plans to address these skills.

Students in a fifth-grade class are beginning a long-term social studies project that has several components. The teacher recognizes that students may find it challenging to manage a complex project and has provided the class with a calendar marked with deadlines for each component. The teacher can best help students manage the project effectively by using which of the following additional strategies? A. creating a class checklist of students' names and the project components so that they can mark their progress in completing the project B. reviewing important project dates with students at the beginning and end of each week C. providing individual students with a personal project folder for organizing their work in progress D. meeting regularly with individual students throughout the project to help them review their progress and set short-term goals for completing their work

D. meeting regularly with individual students throughout the project to help them review their progress and set short-term goals for completing their work This questions requires the examinee to demonstrate knowledge of strategies for promoting students' organizational and time-management skills and sense of responsibility for their won learning. Meeting regularly with students allows the teacher to guide students in areas such as estimating accurately how long a particular task is likely to take and to help them identify strategies for accomplishing particular aspects of the project. With help in setting short-term goals, students are better able to pace their learning so that they are not rushing and feeling pressured as each deadline approaches. This helps them gain a sense of control over their own learning.

When students in a third-grade class come to the teacher with content-related questions such as "How long does a goldfish live?" or "What is the deepest ocean in the world?" the teacher does not immediately answer their questions. Instead, the teacher will look up the answers online or in a reference book. The teacher's approach is likely to benefit student learning primarily by: A. communicating to students the teacher's respect for their questions. B. encouraging students to try to locate answers to their content questions independently. C. helping students develop proficiency in using resources in different formats. D. modeling for students the use of information literacy skills to pursue content knowledge.

D. modeling for students the use of information literacy skills to pursue content knowledge. This question requires the examinee to demonstrate knowledge of information literacy, and the ability to promote students' knowledge of and model and facilitate students' use of the tools, practices, and opportunities of the information age. Modeling information-seeking behaviors and the use of information resources supports students in adopting these behaviors and skills themselves. As the teacher models locating specific information, the students not only learn more about where to find information, but also how to use various resources to address information needs.

Elementary school teachers can best support students in handling difficult social situations by engaging them regularly in activities designed to promote which of the following abilities? A. creative thinking B. problem-centered coping C. analogical thinking D. perspective taking

D. perspective taking Definition: the ability to view a situation from another person's point of view and consider their thoughts and feelings.

During the concrete operational stage of cognitive development as described by Piaget, students benefit from frequent opportunities to manipulate and test objects primarily becasue students at this stage: A. understand concepts more fully when learning tasks include physical activity. B. require scaffolding in the use of visualization as a learning strategy. C. construct knowledge most effectively through a process of trial and error. D. possess a system of thinking that is still tied to physical reality.

D. possess a system of thinking that is still tied to physical reality.

Which of the following approaches is likely to be most effective in facilitating kindergarten students' knowledge construction? A. implementing a system of rewards for class mastery of targeted academic skills B. emphasizing the use of small-group learning activities over independent activities C. designing open-ended experiences that focus primarily on creative thinking D. providing direct interactions and experiences with real objects whenever possible

D. providing direct interactions and experiences with real objects whenever possible

Compared with a classroom assessment system that focuses primarily on test grades, a system that includes multiple forms of data (e.g., anecdotal records, projects) is likely to have which of the following advantages? A. facilitating students' ability to master instructional content at a faster pace B. allowing the teacher to provide students with more accurate feedback on their performance C. creating classroom learning environment that promotes ongoing inquiry D. providing the teacher with a broader base of evidence on which to base instructional decisions

D. providing the teacher with a broader base of evidence on which to base instructional decisions This question requires the examinee to demonstrate knowledge of the purposes of assessment, the relationship between assessment and instruction, and the importance of using a systematic and comprehensive approach to assessment. Students possess varied learning styles and preferences. For example, they may perform better on a performance-based assessment than on an essay assessment. Gathering multiple forms of performance data gives the students more avenues for demonstrating their learning and provides the teacher with a truer picture of their strengths and needs. The teacher can then use this information to design effective instruction for promoting students' continued growth as learners.

A sixth-grade science teacher has been experiencing ongoing difficulty in running short of class time before lesson objectives have been met. The teacher is concerned that students will be unable to achieve all of the sixth-grade science standards if this continues. Which of the following strategies would be most effective in helping the teacher address this problem? A. searching online for science lessons that can be completed in a shorter time period B. assigning students responsibility on a rotating basis for monitoring remaining class time C. extending the length of students' homework assignments to address content not taught in class D. reflecting on instructional plans to identify areas in which pacing can be adjusted

D. reflection on instructional plans to identify areas in which pacing can be adjusted

At the end of a unit covering the Louisiana Purchase, a teacher would like to have a review day before the test. Rather than giving direct instruction, she wants students to work together to review. Which of these methods should be included in her lesson plan? A. the teacher gives a lecture and has stopping points for students to "think-pair-share" B. a review packet is completed by students in groups C. a video over the Louisiana purchase is shown D. students work in groups to create a concept map over the unit

D. students work in groups to create a concept map over the unit This method allows students to discuss concepts and relate new knowledge to prior knowledge.

In March, a fifth-grade teacher is organized a gallery walk, or student rotation around the room, for students to view and comment on the class' past work that hangs on the walls and in mobiles. Which of the following would this best promote? A. the students' self-esteem B. student comprehension and summarization of topic C. class bonding D. students' reflection on past activities and concepts

D. students' reflection on past activities and concepts The viewing and discussing of the student work helps students reflect on their past activities and concepts.

A fourth-grade teacher observes that one student in the class demonstrates exceptional abilities in the area of written expression as evidenced by the quality of assigned essays in multiple content areas. However, the student is very shy and has difficulty expressing ideas during discussions. The student shows depth of thinking and a strong grasp of concepts, and the teacher believes that the student's ideas would enhance the discussions. Which of the following actions by the teacher would best help the student begin to overcome this issue? A. assigning the student a peer partner who is comfortable with the discussion format to relate the student's comments to the group B. speaking to the class when the student is not present and asking for their help in encouraging the student's participation in discussions C. giving the student responsibility for writing down the main points of the discussion for the class instead of actively participating D. suggesting that the student write down comments about a discussion topic in advance and read the comments aloud when appropriate

D. suggesting that the student write down comments about a discussion topic in advance and read the comments aloud when appropriate

At the beginning of a unit on transportation, a first-grade teacher leads students in listing transportation-related questions they want to answer. The teacher writes each question on a sentence strip and posts it on a bulletin board. Throughout the unit, the teacher encourages students to investigate these questions whenever they have free time and post answers on the bulletin board for the class. This practice is likely to be most effective in helping the teacher achieve which of the following goals? A. promoting the development of students' written communication skills B. supporting students in forming positive academic self-concepts C. promoting the development of students' ability to read for information D. supporting students in becoming independent thinkers and learners

D. supporting students in becoming independent thinkers and learners

A second-grade teacher has been conducting systematic observations of a student's behavior during daily recess and is documenting these observations in a narrative form. A representative sample of one of the teacher's narratives is shown below. Student: Jake R. Date: Monday, May 4 Jake is playing with a soccer ball with Michael, Jamil, and Tony. Jake is obviously in charge of the game. Tony tries to capture the ball while Jake is bouncing it on his head. When Jake drops the ball, he is very angry, immediately getting in Tony's face and complaining, "You made me mess up!" Michael and Jamil calmly try to intervene. Jamil says, "You're not sharing with the rest of us." Jake responds with more bullying behavior, including minor shoving, and threatens to never let any of them use the soccer ball again. The most significant problem with this documentation is that the teacher: A. does not provide background information about the students. B. neglects to state the purpose of the observation. C. identifies other students involved in this incident by name. D. uses subjective and judgmental language.

D. uses subjective and judgmental language. This question requires the examinee to demonstrate knowledge of systematic observation and documentation, and the ability to use these processes to gain insight into children's development, interactions, strengths, and needs. The process of systematic observation and documentation requires the observer to note only what is actually observed without making assumptions about the students' emotions or motivations. When the observer eliminates personal bias or interpretations and focuses on concrete actions, the observation data will be more valid.

During center time, a kindergarten teacher observes a student marking in a book in the reading center. Which of the following teacher responses to the student's behavior best illustrates the principles of positive guidance? A. "You seem to be having trouble following the rules in the reading center. Put the books away and go to the block center for the rest of this week." B. "This is an inappropriate way to use books. If you ruin center materials, your classmates won't like you very much. Do you understand?" C. "When you mark in a book, your classmates won't be able to enjoy it. You need to get an eraser and erase the pencil marks. Would you like some help?" D. "Marking in books is not allowed! The next time I see you marking in a book, you will have to sit in the time-out chair."

This question requires the examinee to demonstrate knowledge of developmentally appropriate classroom management approaches and positive guidance techniques. One of the principles of positive guidance is providing the student with a rationale for the rule in question, in this case not marking in books, followed by a logical consequence for not following the rule. Having the student erase the marks promotes accountability for behavior and offering assistance helps communicate to the student that it is the behavior that is unacceptable not the student.


Set pelajaran terkait

Copyright Law: 12 Dos and Dont's

View Set

Medications specific to maternal/newborn nursing

View Set

Ap pyschology unit 5 practice test

View Set

Random assignment evaluation studies

View Set

Ch. 2 (Managing public issues and stakeholder relationships)

View Set

Speech 1110- Final- Noelle Anderson

View Set

Chapter 39 Fluid, Electrolyte, and Acid-Base Balance foundation

View Set

ARRT Test Prep, LIMITED X-RAY CORE

View Set